What is Inequality: Definition and 1000 Discussions

In mathematics, the triangle inequality states that for any triangle, the sum of the lengths of any two sides must be greater than or equal to the length of the remaining side. This statement permits the inclusion of degenerate triangles, but some authors, especially those writing about elementary geometry, will exclude this possibility, thus leaving out the possibility of equality. If x, y, and z are the lengths of the sides of the triangle, with no side being greater than z, then the triangle inequality states that




z

x
+
y
,


{\displaystyle z\leq x+y,}
with equality only in the degenerate case of a triangle with zero area.
In Euclidean geometry and some other geometries, the triangle inequality is a theorem about distances, and it is written using vectors and vector lengths (norms):






x

+

y





x


+


y


,


{\displaystyle \|\mathbf {x} +\mathbf {y} \|\leq \|\mathbf {x} \|+\|\mathbf {y} \|,}
where the length z of the third side has been replaced by the vector sum x + y. When x and y are real numbers, they can be viewed as vectors in R1, and the triangle inequality expresses a relationship between absolute values.
In Euclidean geometry, for right triangles the triangle inequality is a consequence of the Pythagorean theorem, and for general triangles, a consequence of the law of cosines, although it may be proven without these theorems. The inequality can be viewed intuitively in either R2 or R3. The figure at the right shows three examples beginning with clear inequality (top) and approaching equality (bottom). In the Euclidean case, equality occurs only if the triangle has a 180° angle and two 0° angles, making the three vertices collinear, as shown in the bottom example. Thus, in Euclidean geometry, the shortest distance between two points is a straight line.
In spherical geometry, the shortest distance between two points is an arc of a great circle, but the triangle inequality holds provided the restriction is made that the distance between two points on a sphere is the length of a minor spherical line segment (that is, one with central angle in [0, π]) with those endpoints.The triangle inequality is a defining property of norms and measures of distance. This property must be established as a theorem for any function proposed for such purposes for each particular space: for example, spaces such as the real numbers, Euclidean spaces, the Lp spaces (p ≥ 1), and inner product spaces.

View More On Wikipedia.org
  1. jk22

    B Is Bell's inequality Lorentz invariant?

    I browsed the net and found : https://arxiv.org/abs/quant-ph/0408127 It is said the value of Bell's operator depends on the speed, so how can it be Lorentz invariant ?
  2. B

    B Complex number inequality question

    Z can be any point on the argand diagram so if z molous is less than 2 , is that somehow giving us the distance from origin? But how i assumed mod sign only makes things positive therefore its not sqrt( (x+yi)^2 ) = distance ??
  3. S

    MHB Proving Inequality: $\frac{1}{ab+bc+ca}\geq\frac{27}{2(a+b+c)^2}$

    Prove: $\frac{1}{b(a+b)}+\frac{1}{c(b+c)}+\frac{1}{a(c+a)}\geq\frac{27}{2(a+b+c)^2}$ where a,b,c are positives
  4. J

    I Need help with a proof involving points on a quadratic

    Summary: Given three points on a positive definite quadratic line, I need to prove that the middle point is never higher than at least one of the other two. I am struggling to write a proof down for something. It's obvious when looking at it graphically, but I don't know how to write the...
  5. S

    MHB High school inequality 2 |x^2y-a^2b|<A

    Given A>0, find a B>0 such that : for all x,y,a,b : if 0<x<1,0<y<1,0<a<1,0<b<1,and |x-a|<B,|y-b|<B,then |x^2y-a^2b|<A
  6. S

    MHB High school inequality |(√(sinx)+1)^2−(√(sina)+1)^2|<b

    Given 0<a<π/2 , b>0 find a c>0 such that : for all ,x : if 0<x<π/2 and |x-a|<c ,then |(\sqrt sinx +1)^2-(\sqrt sin a +1)^2|<b
  7. chwala

    Solve for x in this inequality problem

    ##7x+12≥3x## ##7x−3x≥−12## ##4x≥−12## → ##x≥−3## or ## 12/x≥-4## but by substituting say## x=−4## we see that it also satisfies the equation implying that my solution may not be correct.
  8. G

    Prove Inequality for Non-increasing Function g(x): Harald Cramer Ex. 4 pg 256

    I have to prove that, for a non-increasing function ##g(x)## the following inequality is true: $$k^2\int_k^\infty g(x) dx\leq\frac{4}{9}\int_0^{\infty}x^2g(x) dx$$ This exercise is from the book Mathematical methods of statistics by Harald Cramer, ex. 4 pg 256 Following the instructions of the...
  9. N

    I Thermal interpretation and Bell's inequality

    Concerning p.198 of Bell's famous 1964 paper http://cds.cern.ch/record/111654/files/vol1p195-200_001.pdf How does TI explain Bell's move from the first equation to the second equation? Under TI, what is the physical significance of this move? Thank you.
  10. karush

    MHB Inequality solve (x+1)/6<x-(3x-2)/4

    Ok a student sent this to me yesterday so want to answer without too many steps I think the first thing to do is multiply every term by 12 $2(x+1)<12x-3(3x-2)$ Expanding $2x+2<12x-9x+6$
  11. G

    MHB Solving Inequality 4x-12≤6x+20

    Hello, I'm working on solving linear equalities (with equations) and can anyone help with the below question. I know the answer is -16, but I can't figure out the steps that gets it to this. 4x-12≤6x+20 Once I've evened out the x's on both sides and got this to 2x, I'm then left with -12...
  12. A

    A Deriving an inequality from a paper

    Hi, I am studying a paper by Yann Bugeaud: http://irma.math.unistra.fr/~bugeaud/travaux/ConfMumbaidef.pdf on page 13 there is an inequality (16) as given below- which is obtained from - , on page 12. How the inequality (16) is derived? I couldn't figure it out. However one of my...
  13. I

    I Exploring Particle Spin and Bell's Inequality

    I know there are numerous threads on this and I have read quite a bit such as EPR and Bell's inequality. I hope I can ask this the right way: A particle has 0 spin and gives off two children particles with spins -1/2 and +1/2 (we don't know which is which yet, or they have to end up this way...
  14. SamRoss

    I Necessity of absolute value in Cauchy Schwarz inequality

    Reading The Theoretical Minimum by Susskind and Friedman. They state the following... $$\left|X\right|=\sqrt {\langle X|X \rangle}\\ \left|Y\right|=\sqrt {\langle Y|Y \rangle}\\ \left|X+Y\right|=\sqrt {\left({\left<X\right|+\left<Y\right|}\right)\left({\left|X\right>+\left|Y\right>}\right)}$$...
  15. S

    Lagrange error bound inequality for Taylor series of arctan(x)

    The error ##e_{n}(y)## for ##\frac{1}{1-y}## is given by ##\frac{1}{(1-c)^{n+2}}y^{n+1}##. It follows that ##\frac{1}{1+y^2}=t_n(-y^2)+e_n(-y^2)## where ##t_n(y)## is the Taylor polynomial of ##\frac{1}{1-y}##. Taking the definite integral from 0 to ##x## on both sides yields that...
  16. Y

    MHB Write an inequality that describes the region where the grass has been planted

    I do not know how to start thinking about part a and then got even more confused when I saw the answer be: P<x^2-5x. I ask for your guidance please.
  17. M

    How can the inequality cosx ≥ (1-x^2/2) be proven?

    How can the inequality ##cosx \ge(1-x^2/2)## be proved? Would you please explain how to prove this inequality? This is the only equation that I could think of. ##1\ge cosx \ge 0## but I cannot use it here. Source: Thomas's Calculus, this is from an integration question there. Thank you.
  18. lfdahl

    MHB Prove the inequality (a^3-c^3)/3≥abc((a-b)/c+(b-c)/a)

    Let $a, b$ and $c$ be non-zero real numbers, and let $a\ge b \ge c$. Prove the inequality: $$\frac{a^3-c^3}{3} \ge abc\left(\frac{a-b}{c}+\frac{b-c}{a}\right)$$ When does equality hold? Source: Nordic Math. Contest
  19. anemone

    MHB Inequality involves radical, square and factorial expression 3√{x}+2y+1z^2⩽ 13

    If $x^2+y^2+z^2+xyz=4$ and that $x,\,y,\,x\ge 0$, prove $3!\sqrt{x}+2!y+1!z^2\le 13$.
  20. opus

    Abs Value Inequality with a Squared Term

    Homework Statement ##\left|\left(\frac{x}{2}\right)^2\right| < 1## Homework EquationsThe Attempt at a Solution The absolute value situation is throwing me off for some reason. Would it be correct to split this into two equations? ##-\left(\frac{x^2}{4}\right) > -1## and ##\frac{x^2}{4} < 1##...
  21. V

    MHB Understanding the Inequality for Solving Limits with Exponential Terms

    Hello! $$\lim_{n\rightarrow \infty }\frac{1}{n}ln(a^{\frac{n}{1}}+a^{\frac{n}{2}}+...+a^{\frac{n}{n}} ), \ a>1$$ I solved the limit by using the following inequality: $$a^{n}\leq a^{\frac{n}{1}}+a^{\frac{n}{2}}+...+a^{\frac{n}{n}}\leq n\cdot a^{n}$$ After I applied a $ln$ and $1/n$ I got $lna$...
  22. W

    I Are There Any Theorems Relating Joint Distributions to Marginals?

    Hi all, I was wondering if there exist any theorems that allow one to relate any joint distribution to its marginals in the form of an inequality, whether or not ##X,Y## are independent. For example, is it possible to make a general statement like this? $$f_{XY}(x,y) \geq f_X (x) f_Y(y)$$...
  23. V

    MHB Solutions of an inequality (1-√(1-4x^2)/x < 3

    I need to find the solutions of the following inequation: (1-sqrt(1-4x^2)/x < 3 I put the conditions x different from 0 and 1-4x^2>=0 and I got [-1/2,0)U(0,1/2] which is the right answer but I'm confuse because I usually subtract 3 to get (1-sqrt(1-4x^2)/x - 3 < 0 then, after I made some work...
  24. R

    Prove Bernoulli's inequality

    Homework Statement Prove bernullis inequality: If h>-1 then (1+h)^n ≥ 1+ nhHomework EquationsThe Attempt at a Solution How can I prove something that is false for h =1 n<1 ?
  25. R

    Find all numbers x which satisfy the following inequality

    Homework Statement (1/x) + (1/(1-x)) > 0 Homework EquationsThe Attempt at a Solution 1+x-x/(x-x^2) > 0 1/(x-x^2) > 0 x-x^2 > 0 x> x^2 only occurs when 0<x<1 but in the solutions Spivak tells me "x>1 or 0<x<1"
  26. R

    Is my proof valid for this inequality problem?

    Homework Statement Prove: if 0<a<b then a^(1/n) < b^(1/n) Homework EquationsThe Attempt at a Solution I've already proved a^n < b^n in another problem. So I have Assume a^n < b^n => \sqrt[n^2] a^n < \sqrt[n^2] b^n => a^(1/n) < b^(1/n)
  27. Vital

    B Chebyshev inequality, confidence intervals, etc

    Hello. I am bewildered by so many different notions of probability distribution percentages, i.e. the proportion of values that lie within certain standard deviations from the mean. (1) There is a Chebyshev's inequality: - for any distribution with finite variance, the proportion of the...
  28. A

    MHB Inequality of Cardinality of Sets

    I am working on a proof problem and I would love to know if my proof goes through: If $A, B$ are sets and if $A \subseteq B$, prove that $|A| \le |B|$. Proof: (a) By definition of subset or equal, if $x \in A$ then $x \in B$. However the converse statement if $x \in B$ then $x \in A$ is not...
  29. Math Amateur

    MHB Exploring Remmert's Derivation of Cauchy-Schwartz Inequality

    I am reading Reinhold Remmert's book "Theory of Complex Functions" ... I am focused on Chapter 0: Complex Numbers and Continuous Functions ... and in particular on Section 1.3: Scalar Product and Absolute Value ... ... I need help in order to fully understand Remmert's derivation of the...
  30. A

    Proof of an inequality with natural numbers

    Homework Statement Prove that ##\forall n \in \mathbb{N}## $$\frac{n}{2} < 1 + \frac{1}{2} + \frac{1}{3} + \ldots + \frac{1}{2^n - 1} \leq n \text{ .}$$ Homework Equations Peano axioms and field axioms for real numbers. The Attempt at a Solution Okay so my first assumption was that this part...
  31. A

    Exploring the Intersection of Ellipsoids and Spherical Shells

    Homework Statement I would like to know how the boundary of the inequality change when the origin of the coordinate system changes. Homework Equations The original inequality is[/B] $$ r_0 \le x^2+y^2+z^2 \le R^2$$ I would like to know the boundary of the following term, considering the...
  32. lfdahl

    MHB Inequality - solve in at least two ways

    Find in at least two different ways the smallest $\alpha$, such that \[\sqrt[3]{x}+\sqrt[3]{y} \leq \alpha \sqrt[3]{x+y}\] - for all $x,y \in \mathbb{R}_+$
  33. Y

    MHB Inequality with fractions solve 2/(x^2−1)≤1/(x+1)

    Dear all, I am trying to solve this inequality: \[\frac{2}{x^{2}-1}\leq \frac{1}{x+1}\] I've tried several things, from multiplying both sides by \[(x^{2}-1)^{2}\] finding the common denominator, but didn't get the correct answer, which is: \[2<x<3\] or \[x<-1\]How to you solve this one ?
  34. anemone

    MHB Trigonometric inequality: sin (1/(n+1934))<1/1994

    Find the smallest natural number $n$ for which $\sin \left(\dfrac{1}{n+1934}\right)<\dfrac{1}{1994}$.
  35. D

    I Solving Trig Integrals with Residue Theorem

    Hi. I have looked through an example of working out a trig integral using the residue theorem. The integral is converted into an integral over the unit circle centred at the origin. The singularities are found. One of them is z1 = (-1+(1-a2)1/2)/a It then states that for |a| < 1 , z1 lies...
  36. J

    MHB Understanding the Cauchy-Schwarz Inequality

    I am not sure what examples to give, need help on this. Have attached the theorem as well.
  37. T

    I Prove this inequality involving metrics

    For any metric ##(X, \rho)## and points therein, prove that ##|\rho(x, z) - \rho(y, u)| \leq \rho(x, y) + \rho(z, u)##. I know that this will involve iterated applications of the triangle inequality...but I still need another hint on how to proceed.
  38. evinda

    MHB Eigenvalues are real numbers and satisfy inequality

    Hello! (Wave) Let $A$ be a $n \times n$ complex unitary matrix. I want to show that the eigenvalues $\lambda$ of the matrix $A+A^{\star}$ are real numbers that satisfy the relation $-2 \leq \lambda \leq 2$. I have looked up the definitions and I read that a unitary matrix is a square matrix...
  39. C

    MHB Proving Modulus Inequality for Complex Numbers

    Dear Everyone, I am currently in an Introduction to Complex Analysis; I have a question: Use established properties of moduli to show that when $\left|{z_3}\right|\ne\left|{z_4}\right|$: $\frac{\Re{({z}_{1}+{z}_{2})}}{\left|{z}_{3}+{z}_{4}\right|}\le\frac{\left| {z}_{1} \right|+\left|...
  40. J

    MHB Real Analysis, liminf/limsup inequality

    I am working a bunch of problems for my Real Analysis course.. so I am sure there are more to come. I feel like I may have made this proof too complicated. Is it correct? And if so, is there a simpler method? Problem: Show that $liminfa_n \leq limsupa_n$. Proof: Consider a sequence of real...
  41. navneet9431

    Need help in solving this question about a rational inequality

    Homework Statement Go through question number 4 Homework Equations The Attempt at a Solution See basically the question is asking us to find the range of the given function x/(x^2+x+1). So,I began solving it this way... I am stuck at this step. I asked my friend for a hint and he told me to...
  42. lfdahl

    MHB Prove Inequality: $f(x)$ Continuous Positive $\int_{0}^{1}$

    Let $f$ be a positive and continuous function on the real line which satisfies $f(x + 1) = f(x)$ for all numbers $x$. Prove the inequality: $$\int_{0}^{1}\frac{f(x)}{f(x+\frac{1}{2})} \,dx \geq 1.$$
  43. Pushoam

    Prove Schwarz Inequality for x, y, z in R+

    Homework Statement For x,y,z ## \in \mathbb {R^+} ##, prove that ## \sqrt {x (3 x +y) } + \sqrt {y (3y +z) } + \sqrt {z(3z +x)} \leq ~ 2(x +y+ z) ##Homework Equations The Attempt at a Solution I don't know which inequality among the above two has to be applied. I am trying to solve it by...
  44. Vanadium 50

    Mathematical inequality measures for the social sciences

    This doesn't really fit in any other place. I'm interested in alternatives to the Gini coefficient for studying inequality, such as were discussed in this thread and this one. What do I see as the shortcomings of Gini? By example: in Lower Slobovia, everyone has nothing. It's a perfectly...
  45. E

    I Spivak's proof of Cauchy Schwarz

    I was browsing through Spivak's Calculus book and found in a problem a very simple way to prove the cauchy schwarz inequality. Basically he tells to substitute x=xᵢ/[√(x₁²+x₂²)] and similarly for y (i=1 and 2), put into x^2 + y^2 >= 2xy. Add the two cases and we get the result. The problem is...
  46. Adgorn

    Spivak's "Calculus": AM-GM inequality problem.

    Homework Statement The problem is stated as follows: "The result in Problem 1-7 has an important generalization: If ##a_1,...,a_n≥0##, then the "arithmetic mean" ##A_n=\frac {a_1+...+a_n} {n}## and "geometric mean" ##G_n=\sqrt[n] {a_1...a_n}## Satisfy ##G_n≤A_n## Suppose that ##a_1\lt A_n##...
  47. M

    MHB Show inequality using the mean value theorem

    Hey! :o Let $D=\left \{x=(x_1, x_2)\in \mathbb{R}^2: x_2>\frac{1}{x_1}, \ x_1>0\right \}$. We have the function $f: D\rightarrow \left (0,\frac{\pi}{2}\right )$ with $f(x)=\arctan \left (\frac{x_2}{x_1}\right )$. I want to show using the mean value theorem in $\mathbb{R}^2$ that for all...
  48. L

    I Generalizing the Bell Inequality for Arbitrary Measurement Axes

    EDIT: I realize now that I have fundamentally misunderstood a crucial aspect of deriving the Bell inequality for this case which is the existence of the third axis. The setup of the problem did state that the axes were chosen at random. Therefore I can't just look at the possibility of choosing...
  49. binbagsss

    Triangle Inequality: use to prove convergence

    Homework Statement Attached I understand the first bound but not the second. I am fine with the rest of the derivation that follows after these bounds, Homework Equations I have this as the triangle inequality with a '+' sign enabling me to bound from above: ##|x+y| \leq |x|+|y| ## (1)...
Back
Top